40 POINTS PLEASE HELPPPPPPP ME !

40 POINTS PLEASE HELPPPPPPP ME !

Answers

Answer 1

Answer:

75/100

=15/20

3/4

The probability is3/4

Step-by-step explanation:

Answer 2

Answer:

3/4

Step-by-step explanation:

Hope this helped have an amazing day!


Related Questions

HELP PLEASE!!!!

Review the function equation below.
y = −9x + 3
Enter the slope of the function in the box.

Answers

-9 is the answer

your welcome

Write an equation of the line that passes through (-1,4) and is perpendicular to the line shown.

Yeah my math class is coocoo, and im stuck on this. Im slow. Help me :(

Answers

Answer:

y=4x+4

Step-by-step explanation:

i was learning this not too long ago so i hope this helps y=mx+b     aka slope is 4 and then for b you put the y intercept aka 4  and then the x and y factors you csn leave blank cause you dont have them

An equation of the line that passes through (-1,4) and is perpendicular to the line given in the graph is y=3x+7.

The coordinates from the line on the graph is (-3, 4) and (3, 2).

What is slope of a line?

The slope of the line is the ratio of the rise to the run, or rise divided by the run. It describes the steepness of line in the coordinate plane.

The formula to find the slope of a line is slope = (y2-y1)/(x2-x1)

Now, slope =(2-4)/(3+3)

= -2/6

= -1/3

Substitute m=-1/3 and (x, y)=(-3, 4) in y=mx+c, we get

4=-1/3 (-3)+c

⇒ c=3

The slope of a line perpendicular to given line is m1=-1/m2

So, slope =3

An equation of the line that passes through (-1,4)

Now, 4=3(-1)+c

c=7

Put m=3 and c=7 in y=mx+c, we get

y=3x+7

Therefore, an equation of the line that passes through (-1,4) and is perpendicular to the line given in the graph is y=3x+7.

To learn more about the slope of a line visit:

https://brainly.com/question/14511992.

#SPJ2

What is the vertex for the graph shown below?
a. (0,0)
b. v=1
c. v=5
d. (5,1)

Answers

Answer:

D. (5,1)

Step-by-step explanation:

The vertex of the graph is V ( 5 , 1 ).

Given data:

The vertex of a function is a point on the graph where the function reaches its highest or lowest point, depending on whether the function is concave up or concave down.

The vertex represents the minimum or maximum point of the function and provides information about the axis of symmetry of the graph. If the function opens upward (concave up), the vertex is the lowest point (minimum).

So, the highest point in the graph where the function reaches the minimum value is when x = 5

So, the value of y = 1

Hence , the vertex is V ( 5 , 1 ).

To learn more about equation of graph of polynomials click :

https://brainly.com/question/16957172

#SPJ2

Please help I really need it!! A student found out that 8% of the students in his school are left-handed.If there are 114 lefties, how many students are in his school?

Answers

Answer:

x = 1425

Step-by-step explanation:

Given that,

A student found out that 8% of the students in his school are left-handed.

Total no of lefties = 114

We need to find the total number of students in the school. Let they are be x.

ATQ,

8% of x = 114

[tex]\dfrac{8x}{100}=114\\\\x=\dfrac{114\times 100}{8}\\\\=1425[/tex]

Hence, there are 1425 students in the school.

the length of the bos is 2 1/2m, it is 1 9/16 times its width. What is the height of the box if its volume is 12 3/4m^3

Answers

Answer:

h=1.3056 m

Step-by-step explanation:

L=2.5m

w=L(1 + 9/16)

w=2.5(1 + 9/16)=[tex]\frac{125}{32}[/tex] m=3 [tex]\frac{29}{32}[/tex]m

v=Lwh

12.75m³=(2.5m)([tex]\frac{125}{32}[/tex]m)h

12.75m³=(9.765625m²)h

[tex]\frac{12.75m^{3} }{9.765625m^{2} }[/tex]=h

h=1.3056 m

Answer:

3 3/16

Step-by-step explanation:

If:

Length= 2 1/2

Width=2 1/2 divided by 1 9/16= 1 3/5

Volume=12 3/4

So then, 2 1/2 times 1 3/5=4

12 3/4 divided by 4= 3 3/16

Done!

BASED ON THE GRAPH, WHICH STATEMENT IS CORRECT?

Answers

Answer:

i would go for a but im not really sure. sorry if incorrect

Step-by-step explanation:

You are traveling down a highway averaging 55 mph. How far would you have traveled after 3 hours and 30 minutes? (Round your answer to the nearest mile)​

Answers

Answer:

193 miles

Step-by-step explanation:

55 x 3.5 = 192.5 round up to 193

Answer:

193 miles I think

Step-by-step explanation:

Hope this helped have an amazing day!

Find the midpoint of segment PQ.

Answers

Answer:

[tex](2,0)[/tex]

General Formulas and Concepts:

Pre-Algebra

Order of Operations: BPEMDAS

Brackets Parenthesis Exponents Multiplication Division Addition Subtraction Left to Right

Algebra I

Midpoint Formula: [tex](\frac{x_1+x_2}{2},\frac{y_1+y_2}{2})[/tex]

Step-by-step explanation:

Step 1: Define

Find endpoints from graph.

Point P (-4, 6)

Point Q (8, -6)

Step 2: Find Midpoint

Simply plug in your coordinates into the midpoint formula to find midpoint .

Substitute [MF]:                    [tex](\frac{-4+8}{2},\frac{6-6}{2})[/tex]Add/Subtract:                       [tex](\frac{4}{2},\frac{0}{2})[/tex]Divide:                                  [tex](2,0)[/tex]

There are 15 female performers in a dance recital. The ratio of men to women is 4:5. How many men are in the dance recital?

Answers

Answer:

12 Men

Step-by-step explanation:

Since the ratio is 4:5 (men:women) 15/5 = 3, so you use 3 as your multiplier and write 3 x 4 = 12 :D

The following cards are used in a game.If each of the cards is turned over and shuffled

Answers

Answer:

than that means the cards are not turned over

im sorry i didnt really get you question

Step-by-step explanation:

There is 1/2 of a greater chance in drawing a spade over drawing a 7

How to determine the probability?

From the complete question added as an attachment, we have the following parameters:

n(Spade) = 4n(7) = 1Total = 6

The probability of getting a spade is:

P(Spade) = 4/6

The probability of getting a 7 is:

P(7) = 1/6

The difference between both probabilities is:

Difference = 4/6 - 1/6

Evaluate

Difference = 3/6

Simplify

Difference = 1/2

Hence, there is 1/2 of a greater chance in drawing a spade over drawing a 7

Note that the question is incomplete, and the missing part of the question can be found online

Read more about probability at:

https://brainly.com/question/251701

#SPJ2

teddy is skiing down a hill from the top to the bottom. the distance from the top of the hill to the bottom is 565 yards. how far will he have to ski before he reaches the midpoint of the hill​

Answers

Answer:

282.5 yards.

Step-by-step explanation:

565 divided by 2 is 282.5.

Just read and answer me

Answers

Answer:

the picture isnt showing up on mines

There are 82 chairs at a birthday party. The chairs need to be lined up in equal-sized rows. How many possible ways can the chairs be lined up

Answers

Answer:

2 rows of 41

Step-by-step explanation:

i had the same question

The number of possible ways can the chairs be lined up is 2 rows of 41.

Given that,

There are 82 chairs at a birthday party. The chairs need to be lined up in equal-sized rows.

Based on the above information, the calculation is as follows:

[tex]= 2 \times 41[/tex]

= 82 chairs

Here 2 denotes the rows.

Therefore we can conclude that the number of possible ways can the chairs be lined up is 2 rows of 41.

Learn more: brainly.com/question/13549064

please help me ASAP ε=ε=ᕕ(°□°)ᕗ​

Answers

Step-by-step explanation:

24cm³ = 0.024dm³

Moles of CO2 = 0.024dm³ / (24dm³/mol) = 0.001mol

Hence 0.001mol of XCO3 is required.

Find the next three terms:
5, 8, 11, 14...

Answers

Answer:

next 3 terms are 17,20,23

Step-by-step explanation:

to get from 5 to 8 you add 3

14+3=17

17+3=20

20+3=23

Answer:

17, 20, 23

Step-by-step explanation:

The pattern is that 3 is added every time.

5 + 3 = 8, 8 + 3 = 11, 11 + 3 = 14.

That means that the next terms will be

14 + 3 = 17

17 + 3 = 20

20 + 3 = 23

Find the value of the expression.
z + y2
for y = 6 and z = 1

Answers

If if y=6 and z=1 then the equation would be 1+6(2) and that ends up to be 13

Answer:

If this is the question then tis is the answer. Thank you. Hope I was of help.

Need help
Question in the screenshot below
NO spam
Please show your work

Answers

Answer:   x = 6

======================================================

Work Shown:

[tex]\log_{4}(x+10)+\log_{4}(x-2)=\log_{4}(64)\\\\\log_{4}\left((x+10)(x-2)\right)=\log_{4}(64)\\\\(x+10)(x-2)=64\\\\x^2-2x+10x-20=64\\\\x^2-2x+10x-20-64=0\\\\[/tex]

[tex]x^2+8x-84=0\\\\(x+14)(x-6)=0\\\\x+14=0 \ \text{ or } \ x-6=0\\\\x=-14 \ \text{ or } \ x=6\\\\[/tex]

Those are the possible solutions, but plugging x = -14 back into the original equation will lead to an error. So we rule x = -14 out

x = 6 works as a solution however

What is the measure of A?

Answers

The answer is 35 cause first u multiply 6+5 which is 30 then add 5 which is 35

i like yo cut g'slol

Answers

I like urs tooth u ritutit twas tthe was the one that had a baby boy and a girl that had

please help!!!!!!!!!

Answers

Answer:

Using only dilation

Step-by-step explanation:

This is because dilation is to make bigger or smaller.

Answer:

c

Step-by-step explanation:

dilation is when you shrink the square, but maintain similarity between the four side lengths. Theoretically, you can shrink square EFGH to the size of ABCD

Em um show foram 150 mil pessoas. Sendo 1/4 de pessoas estavam no camarote rosa, 1/5 no camarote lilas, 2/5 no camarote Amarelo, e 15% na pista. Qual foi o local que obteve maior número de pessoas? * 1 ponto a) Camarote Rosa b) Camarote Lilas c) Camarote Amarelo d) Pista

Answers

Responda:

camarote Amarelo

Explicação passo a passo:

Dado que:

Número total de pessoas = 150.000

Caixa rosa: 1/4 * 150.000 = 37.500

Caixa Lilax: 1/5 * 150.000 = 30.000

Caixa amarela: 2/5 * 150.000 = 60.000

Faixa = 0,15 * 150.000 = 22.500

Portanto, a caixa amarela tem mais gente

Given the equation: y=2x-5 , is your y-intercept below or
above the x-axis?

Answers

Answer:

Below

Step-by-step explanation:

The equation for slopes are y=mx+b. m = slope, and b = y-intercept.

Since b is negative the y-intercept will be below the x-axis.

In 1912, the U.S. government began issuing licenses to radio stations. Each station was given a unique three-letter call sign. Radio stations in the western United States were given call signs starting with K. Stations in the east were given call signs starting with W. For example, the radio station with the call sign ABC is KABC in California, and WABC in Georgia.



a. How many different three-letter call signs are there?

Hint: don’t worry about the K or W – just find the number of ways to arrange any 3 letters



b. How many different three-letter call signs are there where the first letter is a D?

Hint: again, the K and W don’t matter



c. In a random selection of radio station call signs, what is the probability of selecting a radio station with a call sign that starts with the letter D?

Answers

Answer:

A would make sense

Step-by-step explanation:

(Sigma)20 4(8/9)^n-1=?
1

Answers

Convert to a simplified fraction.

20 amgiS ⋅ 9n / 2^2+3n − 9^n

Move 20 to the left of Sigma.

20 Sigma

Eric conducted a survey of the cars in four parking lots and wrote the following observations:

15 of the 36 cars in lot 1 were blue
43 of the 92 cars in lot 2 were blue
12 of the 30 cars in lot 3 were blue
32 of the 52 cars in lot 4 were blue

Based on the survey, which parking lot had the greatest percentage of blue cars?


explain your answer

Answers

Answer:

Lot 4 since 61.5% is the highest percentage of blue cars of all lots.

Step-by-step explanation:

Find the percentage that each number of blue cars is in its parking lot.

percent = part/whole * 100%

15 of the 36 cars in lot 1 were blue

15/36 * 100% = 41.667%

43 of the 92 cars in lot 2 were blue

43/92 * 100% = 46.739%

12 of the 30 cars in lot 3 were blue

12/30 * 100% = 40%

32 of the 52 cars in lot 4 were blue

32/52 * 100% = 61.538%

Answer: Lot 4 since 61.5% is the highest percentage of blue cars of all lots.

Answer:

Parking Lot 4

Step-by-step explanation:

15÷36 x 100 = 41.666%

43÷ 92 x 100 = 46.739%

12 ÷ 30 x 100 = 40%

32÷ 52 x 100 = 61%

61% is the largest percentage out of all of them, therefore Parking Lot 4 ( 32 out the 52 cars) is the Parking Lot that has the greatest percentage of blue cars!

CAN I PLEASE BE BRAINLIST?

Given the parallelogram below, find the value of angle 1

Answers

who even knows now days

6. Michelle orders cupcakes that come in a carboard box. The box has the dimensions shown. What is the perimeter of the base? b. What is the height of the box? What is the area of the base
9in,10in,11in​

Answers

Can u send a pic of the square kinda hard without it

iogmkiuAnswer:

Step-by-step explanation:

gjkih

Based on this model, a school with a 53% chance of winning their first game would have what rank? Round your answer to the nearest whole number.

Answers

Answer:

8

Step-by-step explanation:

50 is at rank 8.5, but since its a 53% chance. it would be below the .5 area and round to 8

Determine whether or not the equation shows a proportional relationship. If it does, identify the unit rate (constant of proportionality).
y =25x, a = 5x + 5, m = 105.25n

Answers

Answer:

y = 25·x shows a proportional relationship

a = 5·x + 5 does not show a proportional relationship

m = 105.25·n shows a proportional relationship

Step-by-step explanation:

An equation that shows a proportional relationship is of the form, y = kx

Where;

k = The constant of proportionality = y/x

Therefore, the equation, y = 25·x shows a proportional relationship

The constant of proportionality = k = y/x = 25

Similarly, the equation m = 105.25·n shows a proportional relationship

The constant of proportionality = k = m/n = 105.25

When the equation shows a proportional relationship, the graph of the equation passes through the origin, therefore, the equation of the form, y = m·x + c, where c = the y-intercept > 0, the equation does not show a proportional relationship

The equation a = 5·x + 5, which is of the form, y = m·x + c, with c = 5 > 0, therefore, does not show a proportional relationship.

Therefore;

y = 25·x shows a proportional relationship

m = 105.25·n shows a proportional relationship

a = 5·x + 5 does not show a proportional relationship.

What is the slope of 7,-2 and -3,3

Answers

Answer:

-1/2

Step-by-step explanation:

y2-y1/x2-x1

3-(-2)/ -3-7

5/-10

= -1/2

Other Questions
fing the sum of the interior angle measures of the polygon If bids on keywords in a bid strategy are manually overwritten, how long will it take for the bid strategy to resume bidding on those keywords Which of the following are common decomposers? (Select all that apply) fungi, ferns, bacteria, viruses, grasses To be three dimensional, a piece of art must havecolordeptha flat surfacea human actor Which term is used to describe the cubist viewpoint where a subject is broken down to take the view away from reality?A realismB unrealC hyperrealD realing A saturated hydrocarbon having molecular formula CnH2n+2 diffuses through a porous membrane twice as fast as sulphur dioxide. Calculate the volume occupied by the hydrocarbon at27C and 2atm ? Help me fast ! how were Europeans were able to gain power? how did Europeans take advantage of other people in the world during the Age of Exploration? lastly, what impact did European exploration have on the rest of the world? what is the volume of a regular octahedron inside a cube with the length of its sides measuring 4 centimeters Which of the following quotes best represents the theme of gender in the novel?None of the Iraqi men seemed to take notice of Marla, but the women did, and I saw one of them smiling.""We want freedom, we want love, we want a chance to go to heaven when the time comes."we will be able to divert men away from that operation to providing security for your people."You're not allowed to treat Iraqis." Write an equation in slope-intercept form for the line parallel to y=3x-2 that passes through the point (5,-2) Zach created a blueprint using only similar parallelograms. Which statement about the parallelograms must be true? A) The parallelograms all have congruent sides and angles. B) The parallelograms have corresponding angles that are not congruent. C) The parallelograms have proportional corresponding sides. D) The parallelograms have corresponding sides that are congruent. Either of the options ____ acceptable to me.areis A bicyclist is riding a constant speed 62.0 seconds for 200. meters what is his average speed The points in the table lie on a line. Find the slope of the line. 3w-5>2w+-7 and 6(c+3) 2+-3 An exoplanet with one half of Earth's mass and 50% of Earth's radius is discovered.What would a space cadet who weighs 800 N on Earth weigh on the exoplanet? A wire 4 1/2 feet long is being cut into smaller pieces of wire, each 11/4 feet long. What is the maximum number of the smaller pieces of wire that can be cut from the original piece of wire? WHO EVER CAN TRANSLATE INTO SPANISH WILL GET BRAINLIEST!!!!!!!!!!!!!!You are at the doctor's office because you are not feeling well. In Spanish, explain to the doctor what's wrong with you. Be sure to use at least 4 complete BEGINNER sentences. what is the most memorable trip of your life? Bart thinks mice exposed to microwaves will get stronger (maybe he's been reading too much Radioactive Man). You decide to perform this experiment by placing 10 mice in a microwave for 10 seconds. He compared these 10 mice to 10 other mice that had not been exposed. His test consisted of a heavy block of wood that blocked the mouse's food. found that 8 out of 10 of the microwave mice were able to push the block. 7 out of 10 of the mice without microwaves were able to do the same.identify1. control group2. independent variable3. dependent variable4. what are some of the mentioned control variables? 5. what should bart's conclusion be?